Time Left - 08:00 mins

Quiz on Data Sufficiency and Verbal Reasoning

Attempt now to get your rank among 877 students!

Question 1

In the question below, a statement is followed by three assumptions numbered I, II and III. An assumption is something supposed or taken for granted. You have to consider the statement and the following assumptions and decide which assumptions are implicit in the statement.
It is believed by many economists that to realize a 7 percent GDP growth rate in India, which is very much attainable, the gross fixed capital formation in the country must increase to 30 percent of GDP from the present level of 28 percent.
I. The target of 7 percent GDP growth is not feasible.
II. GDP growth rate is directly related to capital formation rate.
III. The GDP growth rate in a country is the only indicator of country’s economic development.

Question 2

The statement below is followed by three outcomes numbered I, II and II. An outcome is either a step of administrative decision to be undertaken for improvement, or a follow-up for further action, or natural response by stakeholders, etc. on the basis of the information provided in the statement. Everything mentioned in the statement is to be assumed to be true, on the basis of which the most logically followed course of action has to be decided.
Statement: The city council of Brownwood City has decided to install a plant of mineral water to provide the citizens mineral water bottles at US $1 per bottle s against bottles costing US $ 1.5 being sold by local private playesr.
I. All the local private companies selling bottled water in Brownwood City will have to close their operations.
II. The city council of Brownwood City will have to provide for the losses from this project in its budget.
III. The normal tap water supply of Brownwood City will have no takers and that will have to be discontinued.

Question 3

Sports teams have almost always suffered from a lack of bench strength time and again. They have experimented with various fitness training programs and rotation policies to keep older players on the bench and ready to serve if necessary. Yet, when it comes to scouting for local talent, they have been surprisingly conservative.

Which of the following, if true, most strengthens the argument above?

Question 4

Directions: Read the given statements carefully and answer the questions which follow.
Small grocery shop owners have gone on a day’s strike in order to protest against the entry of a foreign retail store chain in the city.

Which of the following statements can be an effect of the statement given above ?

Question 5

Directions: In each question below is given a statement followed by two assumptions numbered I and II. An assumption is something supposed or taken for granted. You have to consider the statement and the following assumptions and decide which of the assumptions is implicit in the statement.
Statement: The railway authority has announced suspension of movements of train on the main track within the city limit for carrying out major repair works during Saturday and Sunday and advised the commuters to plan their journey accordingly.
Assumptions:
I. The commuters may protest against the decision of the railway authority and may disrupt other transport services.
II. The municipal authority may be able to deploy additional buses during Saturday and Sunday to help the commuters.

Question 6

Direction: The question below consists of a question and two statements numbered I and II given below it. You have to decide whether the data provided in the statements are sufficient to answer the question.
Amongst H, J, K and L sitting around a circular table facing the centre, who sits to the immediate left of H?
I. H sits second to left of L, K is an immediate neighbour of L.
II. J and H are immediate neighbours of each other. L is an immediate neighbour of J.

Question 7

Directions: Each of the questions below consists of a question and two statements numbered I and II given below it. You have to decide whether the data provided in the statements are sufficient to answer the question. Read both the statements
Out of 64 students, 38 play both chess and cricket. How many students play only chess?
I. Out of 64 students, 22 students don't play any game. 4 students play only cricket.
II. Out of 64 students, 20 are girls and 10 of them don't play any game

Question 8

Direction: Each of the questions below consists of a question and two statements numbered I and II are given below it. You have to decide whether the data provided in the statements are sufficient to answer the question.
‘H’ is in which direction with respect to ‘L’?
I. ‘L’ is to the East of ‘M’ which is to North of ‘H’.
II. ‘L’ is to the North of ‘J’ which is to the East of ‘H’.

Question 9

Each of the questions below consists of a question and two statements numbered I and II are given below it. You have to decide whether the data provided in the statements are sufficient to answer the question.
What is Sonal’s rank from the top in a group of 10 students?
I. Meenu’s rank is three ranks above Sonal.
II. Sita is second from the bottom and is three ranks below Sonal.

Question 10

What is Rakesh's age in a group of four members?
I. Rakesh, Vimal and Sandeep are all of the same age.
II. Total age of Vimal, Kunal and Sandeep is 32 years, and Kunal is as old as Vimal and Sandeep together.
  • 877 attempts
  • 13 upvotes
  • 15 comments
May 2PO, Clerk, SO, Insurance